Skip to main content

Die Newton’schen Axiome

  • Chapter
  • First Online:
Theoretische Physik

Kapitelvorwort

Was bedeutet „klassische Mechanik“?

Wie lauten die Newton’schen Axiome?

Was ist ein Inertialsystem?

Wie löst man einfache Bewegungsgleichungen?

Was sind konservative Kräfte?

Was besagt der Energieerhaltungssatz?

Das vorliegende Kapitel liefert eine Einführung in die grundlegenden Konzepte der klassischen, nichtrelativistischen Mechanik. Ein wichtiges Fundament dieser Theorie ist das physikalische Verständnis der Begriffe „Raum“ und „Zeit“, „Körper“ und „Masse“, „Kraft“ und „Inertialsystem“ (Abschn. 1.1). Auf der Basis dieser Größen werden wir die Newton’schen Axiome kennenlernen und erarbeiten (Abschn. 1.2). Sie bestimmen, auf welchen Bahnkurven sich Punktmassen bewegen. Dies wird zunächst anhand eindimensionaler Beispiele verdeutlicht (Abschn. 1.3), was auf den Energiesatz in einer Dimension führt (Abschn. 1.4). Die Verallgemeinerung der Bewegung einer Punktmasse auf drei Dimensionen (Abschn. 1.5) erlaubt uns schließlich den Energiesatz in drei Dimensionen zu formulieren (Abschn. 1.6). Zusätzlich werden wir zahlreiche mathematische Begriffe definieren und Techniken kennenlernen, die ein tieferes Verständnis der Mechanik überhaupt erst ermöglichen.

This is a preview of subscription content, log in via an institution to check access.

Access this chapter

Chapter
USD 29.95
Price excludes VAT (USA)
  • Available as PDF
  • Read on any device
  • Instant download
  • Own it forever
eBook
USD 79.99
Price excludes VAT (USA)
  • Available as EPUB and PDF
  • Read on any device
  • Instant download
  • Own it forever
Hardcover Book
USD 99.99
Price excludes VAT (USA)
  • Durable hardcover edition
  • Dispatched in 3 to 5 business days
  • Free shipping worldwide - see info

Tax calculation will be finalised at checkout

Purchases are for personal use only

Institutional subscriptions

Literatur

Weiterführende Literatur

Download references

Author information

Authors and Affiliations

Authors

Corresponding author

Correspondence to Matthias Bartelmann .

Appendices

Aufgaben

Gelegentlich enthalten die Aufgaben mehr Angaben, als für die Lösung erforderlich sind. Bei einigen anderen dagegen werden Daten aus dem Allgemeinwissen, aus anderen Quellen oder sinnvolle Schätzungen benötigt.

•:

leichte Aufgaben mit wenigen Rechenschritten

••:

mittelschwere Aufgaben, die etwas Denkarbeit und unter Umständen die Kombination verschiedener Konzepte erfordern

•••:

anspruchsvolle Aufgaben, die fortgeschrittene Konzepte (unter Umständen auch aus späteren Kapiteln) oder eigene mathematische Modellbildung benötigen

1.1 •• Vektorrechnung

Das Rechnen mit Vektoren, Skalar‐ und Vektorprodukten soll hier geübt werden.

  1. (a)

    Wir betrachten n Vektoren v i aus einem Vektorraum V. Man kann diese Vektoren verknüpfen, um die Linearkombination

    $$v=\lambda_{1}v_{1}+\ldots+\lambda_{n}v_{n}$$
    (1.172)

    zu erhalten. Hier sind die n Größen \(\lambda_{i}\) Skalare. Die Menge aller dieser darstellbaren Vektoren v bildet einen Untervektorraum U von V. Man bezeichnet U als den von den v i aufgespannten Untervektorraum \(U=\mathrm{span}(v_{i})\). Man nennt die Vektoren v i linear unabhängig , wenn die Gleichung

    $$\lambda_{1}v_{1}+\ldots+\lambda_{n}v_{n}=0$$
    (1.173)

    nur dann erfüllt werden kann, wenn alle Skalare \(\lambda_{i}\) verschwinden: \(\lambda_{i}=0\). In diesem Fall lässt sich ein Vektor v als eindeutige Linearkombination der v i darstellen, d. h., die \(\lambda_{i}\) sind für einen gegebenen Vektor v eindeutig.

    Überprüfen Sie, welche der folgenden Sätze von Vektoren im \({\mathbb{R}}^{3}\) linear unabhängig sind:

    $${\boldsymbol{v}}_{1}=\begin{pmatrix}1\\ 0\\ 0\end{pmatrix},\quad{\boldsymbol{v}}_{2}=\begin{pmatrix}0\\ 1\\ 0\end{pmatrix},\quad{\boldsymbol{v}}_{3}=\begin{pmatrix}0\\ 0\\ 1\end{pmatrix};$$
    (1.174)
    $${\boldsymbol{v}}_{1}=\begin{pmatrix}1\\ 1\\ 1\end{pmatrix},\quad{\boldsymbol{v}}_{2}=\begin{pmatrix}1\\ 1\\ 0\end{pmatrix},\quad{\boldsymbol{v}}_{3}=\begin{pmatrix}1\\ 0\\ 0\end{pmatrix};$$
    (1.175)
    $${\boldsymbol{v}}_{1}=\begin{pmatrix}-3\\ 2\\ 1\end{pmatrix},\quad{\boldsymbol{v}}_{2}=\begin{pmatrix}1\\ 2\\ 3\end{pmatrix},\quad{\boldsymbol{v}}_{3}=\begin{pmatrix}2\\ -1\\ -3\end{pmatrix}.$$
    (1.176)

    Was passiert, wenn im dritten Beispiel der dritte Vektor durch \({\boldsymbol{v}}_{3}=(2,-4,-4)^{\top}\) ersetzt wird?

  2. (b)

    Skalarprodukte können wesentlich allgemeiner definiert werden als das innere Produkt \({\boldsymbol{x}}\cdot{\boldsymbol{y}}=\sum_{i=1}^{3}x_{i}y_{i}\) für Vektoren im \({\mathbb{R}}^{3}\) bezüglich einer Orthonormalbasis. Man betrachte beispielsweise den Vektorraum der Polynome vom Grad N (d. h., die Polynome dürfen höchstens Terme der Ordnung x N enthalten) auf dem Intervall \([-1,1]\). Das Skalarprodukt sei durch

    $$\langle p,q\rangle=\int_{-1}^{1}p(x)q(x)\,{\mathrm{d}}x$$
    (1.177)

    definiert. Welche dieser drei Polynome sind im Sinne dieses Skalarprodukts zueinander orthogonal?

    $$p(x)=x,\quad q(x)=x^{2}-2,\quad r(x)=x^{3}+x.$$
    (1.178)

    Wie könnte man die Polynome normieren?

  3. (c)

    Häufig benötigt man das Spatprodukt , das auch als Mehrfachprodukt bezeichnet wird:

    $$({\boldsymbol{a}}\times{\boldsymbol{b}})\cdot{\boldsymbol{c}}=({\boldsymbol{b}}\times{\boldsymbol{c}})\cdot{\boldsymbol{a}}=({\boldsymbol{c}}\times{\boldsymbol{a}})\cdot{\boldsymbol{b}}=\varepsilon_{ijk}a_{i}b_{j}c_{k}.$$
    (1.179)

    Zeigen Sie, dass es verschwindet, wenn einer der Vektoren als Linearkombination der beiden anderen geschrieben werden kann. Dies ist dann der Fall, wenn alle drei Vektoren in einer Ebene liegen. Die drei Vektoren spannen dann einen Spat mit Volumen null auf.

  4. (d)

    Zeigen Sie die Lagrange‐Identität

    $$({\boldsymbol{a}}\times{\boldsymbol{b}})\cdot({\boldsymbol{c}}\times{\boldsymbol{d}})=({\boldsymbol{a}}\cdot{\boldsymbol{c}})({\boldsymbol{b}}\cdot{\boldsymbol{d}})-({\boldsymbol{a}}\cdot{\boldsymbol{d}})({\boldsymbol{b}}\cdot{\boldsymbol{c}}).$$
    (1.180)

    Verwenden Sie dazu die Levi‐Civita‐ und Kronecker‐Symbole.

1.2 •• Schiefer Wurf

Ein Speerwerfer trainiert für einen Wettkampf. Er überlegt sich, dass es einen optimalen Abwurfwinkel geben muss, um die Reichweite zu maximieren. Dies ist offensichtlich: Wird der Speer waagerecht geworfen, fällt er nach kurzer Flugstrecke auf den Boden. Wirft man ihn fast senkrecht nach oben, ist das Ergebnis ebenfalls enttäuschend. Man bestimme die allgemeine Bahnkurve des reibungsfreien schiefen Wurfes in der x-z-Ebene, wobei die Gravitation in negative z-Richtung wirkt. Die Anfangsbedingungen für den Ort und die Geschwindigkeit sind \(x(0)=x_{0}\), \(z(0)=z_{0}\) und \(v_{x}(0)=v_{x,0}\), \(v_{z}(0)=v_{z,0}\).

  1. (a)

    Man zeige, dass es sich um eine Parabelbahn (Wurfparabel) handelt. Wie hoch liegt der Scheitelpunkt? Wann wird er erreicht?

  2. (b)

    Der Wurfwinkel zwischen Anfangsgeschwindigkeitsvektor und dem Boden (x-Achse) ist α. Stellen Sie zunächst jeweils einen Zusammenhang zwischen dem Betrag der Anfangsgeschwindigkeit, v 0, dem Wurfwinkel α und den Projektionen der Anfangsgeschwindigkeit auf die x- und z-Achsen, \(v_{x,0}\) und \(v_{z,0}\), auf.

  3. (c)

    Leiten Sie eine Gleichung ab, die die Reichweite des Wurfes als Funktion der Anfangsgeschwindigkeit v 0 und des Wurfwinkels α beschreibt. Gehen Sie dabei davon aus, dass sowohl die Abwurf‐ als auch die Endhöhe z 0 sind. Für welchen Wurfwinkel wird die Reichweite bei festem v 0 maximal?

  4. (d)

    Kann ein Fußballtorwart einen Fußball direkt ins gegnerische Tor schießen, wenn der Ball mit \(v_{0}=100\,\mathrm{km}\,\mathrm{h}^{-1}\) aus dem eigenen Torraum abgeschlagen wird? Ein Fußballfeld ist etwa 100 m lang. Reibungseffekte sollen vernachlässigt werden.

1.3 ••• Schiefer Wurf mit Stokes’scher Reibung

Betrachten Sie den schiefen Wurf mit Stokes’scher Reibung (Reibungskoeffizient K). Die Bewegung erfolge in der x-z-Ebene, und die homogene Gravitationskraft zeige in negative z-Richtung. Die Anfangsbedingungen für den Ort und die Geschwindigkeit sind \(x(0)=x_{0}\), \(z(0)=z_{0}\) und \(v_{x}(0)=v_{x,0}\), \(v_{z}(0)=v_{z,0}\).

  1. (a)

    Zeigen Sie, dass die Bewegungsgleichungen entkoppelt sind.

  2. (b)

    Schreiben Sie die allgemeine Lösung der Bewegungsgleichungen für x und z auf. Orientieren Sie sich dabei an den Schritten, die auf (1.43) geführt haben. Für das weitere Vorgehen bietet es sich an, die Endgeschwindigkeit \(v_{\mathrm{E}}\) nicht als Abkürzung einzuführen.

  3. (c)

    Berechnen Sie die allgemeine Bahnkurve \(z(x)\). Dazu muss die Zeit aus der Lösung \(z(t)\) mithilfe von \(x(t)\) eliminiert werden.

  4. (d)

    Zeigen Sie, dass der Grenzfall K = 0 auf die Lösung für die Bahnkurve ohne Reibung aus Aufgabe 2 führt. Achtung: Es ist nicht möglich, einfach K = 0 zu setzen. Stattdessen muss der Logarithmus in \(z(x)\) für kleine K entwickelt werden. Danach kann der Grenzfall \(K\to 0\) durchgeführt werden. Nutzen Sie aus, dass \(\ln(1+x)\approx x-x^{2}/2\) gilt. Warum muss an dieser Stelle die zweite Ordnung der Taylor‐Entwicklung ebenfalls mitgenommen werden?

1.4 •• Lorentz‐Kraft

Auf ein geladenes Teilchen (elektrische Ladung q) wirkt im elektrischen Feld \({\boldsymbol{E}}\) und magnetischen Feld \({\boldsymbol{B}}\) die Lorentz‐Kraft (in SI‐Einheiten)

$${\boldsymbol{F}}=q\left({\boldsymbol{E}}+{\boldsymbol{v}}\times{\boldsymbol{B}}\right).$$
(1.181)

Es soll die Bewegung in einem reinen Magnetfeld, das homogen und zeitunabhängig ist, untersucht werden. Hierzu wird das Koordinatensystem so gelegt, dass

$${\boldsymbol{B}}=B\,{\boldsymbol{\hat{e}}}_{z}$$
(1.182)

gilt. Das elektrische Feld wird vernachlässigt: \({\boldsymbol{E}}=0\).

  1. (a)

    Zeigen Sie ohne weitere Rechnungen, dass der Betrag der Geschwindigkeit des Teilchens konstant ist und dass keine Arbeit am Teilchen verrichtet wird. Die Energie ist somit erhalten und die Kraft konservativ. Greifen Sie dazu auf die Ergebnisse zurück, die in Abschn. 1.5 und 1.6 erarbeitet wurden.

  2. (b)

    Stellen Sie die Bewegungsgleichungen für das Teilchen auf. Die Anfangsbedingungen lauten \({\boldsymbol{x}}(t=0)=0\) und \(\dot{{\boldsymbol{x}}}(t=0)=(0,v_{y,0},v_{z,0})^{\top}\). Die Lösung für die Bewegung entlang der z-Achse kann direkt hingeschrieben werden. Wie lautet sie?

  3. (c)

    Wie lauten die Lösungen der Bewegungsgleichungen entlang der x- und y-Achsen? Um die beiden gekoppelten Gleichungen zu entkoppeln, integrieren Sie zunächst eine der beiden Bewegungsgleichungen und setzen Sie das Ergebnis in die andere ein.

  4. (d)

    Wie lauten der Tangential‐ und der Hauptnormalenvektor der Bahnkurve? Bestimmen Sie den Krümmungsradius.

1.5 •• Differenzialoperatoren

In dieser Aufgabe wird die Anwendung der Differenzialoperatoren vertieft.

  1. (a)

    Berechnen Sie die Rotation einer allgemeinen radialsymmetrischen Zentralkraft:

    $${\boldsymbol{F}}({\boldsymbol{r}})=F(r)\frac{{\boldsymbol{r}}}{r}.$$
    (1.183)

    Verwenden Sie dazu \(r^{2}=x_{1}^{2}+x_{2}^{2}+x_{3}^{2}\). Es bietet sich an, die Rotation mithilfe des Levi‐Civita‐Symbols in Indexschreibweise zu bestimmen.

  2. (b)

    Wie lautet die Rotation der Kraft

    $${\boldsymbol{F}}({\boldsymbol{r}})=\begin{pmatrix}-x_{2}/\varrho^{2}\\ x_{1}/\varrho^{2}\\ 0\end{pmatrix}$$
    (1.184)

    mit \(\varrho^{2}=x_{1}^{2}+x_{2}^{2}\)? Verwenden Sie hier die Schreibweise

    $${\boldsymbol{\nabla}}\times{\boldsymbol{F}}({\boldsymbol{r}})=\varepsilon_{ijk}{\boldsymbol{\hat{e}}}_{i}\partial_{j}F_{k}.$$
    (1.185)

    Berechnen Sie weiterhin das Kurvenintegral über die Kraft entlang des Weges

    $${\boldsymbol{r}}(\varphi)=R\begin{pmatrix}\cos\varphi\\ \sin\varphi\\ 0\end{pmatrix},\quad 0\leq\varphi\leq 2\uppi.$$
    (1.186)

    Was ist die Interpretation dieser Resultate?

  3. (c)

    Gegeben sei ein Skalarfeld \(g({\boldsymbol{x}})\). Zeigen Sie, dass die Richtungsableitung entlang der Achse \({\boldsymbol{\hat{e}}}_{j}\), also \(\partial_{{\boldsymbol{\hat{e}}}_{j}}g({\boldsymbol{x}})\), identisch ist mit der partiellen Ableitung \(\partial_{j}g({\boldsymbol{x}})\).

  4. (d)

    Zeigen Sie, dass die Divergenz eines Rotationsfeldes verschwindet:

    $${\boldsymbol{\nabla}}\cdot\left({\boldsymbol{\nabla}}\times{\boldsymbol{h}}({\boldsymbol{x}})\right)=0,$$
    (1.187)

    wobei \({\boldsymbol{h}}({\boldsymbol{x}})\) ein zweifach differenzierbares Vektorfeld ist.

  5. (e)

    Man berechne die Rotation eines Rotationsfeldes: \({\boldsymbol{\nabla}}\times\left({\boldsymbol{\nabla}}\times{\boldsymbol{h}}({\boldsymbol{x}})\right)\). Stellen Sie das Ergebnis in Komponentenschreibweise dar.

1.6 •• Bewegung im Kraftfeld

Es ist das Kraftfeld

$${\boldsymbol{F}}({\boldsymbol{x}})=C\begin{pmatrix}k_{1}x_{1}x_{2}\\ x_{1}^{2}+k_{2}x_{3}^{2}\\ x_{2}x_{3}\end{pmatrix}$$
(1.188)

gegeben (C > 0 ist eine dimensionsbehaftete Konstante, sodass \({\boldsymbol{F}}\) die Einheit einer Kraft besitzt). Die Parameter k 1 und k 2 sind zunächst offen.

  1. (a)

    Für welchen Wert von k 1 und k 2 besitzt diese Kraft ein Potenzial? Rechnen Sie im Folgenden mit diesen Werten weiter.

  2. (b)

    Berechnen Sie die benötigte Arbeit, um eine Punktmasse m vom Ursprung zum Punkt \({\boldsymbol{a}}=(a_{1},a_{2},a_{3})^{\top}\) zu befördern. Der gewählte Weg spielt dabei keine Rolle. Wählen Sie den direkten Weg entlang einer Geraden. Wann benötigt man für den Transport Energie, wann gewinnt man Energie?

  3. (c)

    Wie lautet das Potenzial? Überprüfen Sie, dass es auf die korrekte Kraft führt.

1.7 ••• Harmonischer Oszillator

Es wird die Lösung der Schwingungsgleichung (1.101) des harmonischen Oszillators gesucht:

$$\ddot{x}+\omega_{0}^{2}x=0,\quad\omega_{0}^{2}=\frac{k}{m}.$$
(1.189)
  1. (a)

    Machen Sie einen Potenzreihenansatz

    $$x(t)=\sum_{j=0}^{\infty}c_{j}(\omega_{0}t)^{j}$$
    (1.190)

    mit konstanten Koeffizienten c j (\(j=1,\ldots,\infty\)). Zeigen Sie, dass zwischen den Koeffizienten der folgende Zusammenhang besteht:

    $$c_{j+2}=-\frac{c_{j}}{(j+2)(j+1)}.$$
    (1.191)

    Dies nennt man eine Rekursionsgleichung für die Koeffizienten c j . Begründen Sie, warum c 0 und c 1 beliebig wählbar sind und wie dies zu interpretieren ist.

  2. (b)

    Zeigen Sie, dass aus der Rekursionsgleichung die explizite Form

    $$c_{j}=\begin{cases}\displaystyle(-1)^{j/2+1}\frac{c_{0}}{j!}&j\;\mathrm{gerade}\\ \displaystyle(-1)^{(j-1)/2}\frac{c_{1}}{j!}&j\;\mathrm{ungerade}\end{cases}$$
    (1.192)

    folgt.

  3. (c)

    Bestimmen Sie die Taylor‐Entwicklung der Funktionen \(\cos(\omega_{0}t)\) und \(\sin(\omega_{0}t)\) um t = 0 bis einschließlich des dritten nichtverschwindenden Terms. Wie lauten die Koeffizienten der Entwicklung? Was folgt daraus?

  4. (d)

    Überprüfen Sie, dass \(x_{1}(t)=\cos(\omega_{0}t)\) und \(x_{2}(t)=\sin(\omega_{0}t)\) die Schwingungsgleichung lösen. Außerdem löst \(A\cos(\omega_{0}t-\phi_{0})\) die Schwingungsgleichung, da es eine Linearkombination der sogenannten Fundamentallösungen \(x_{1}(t)\) und \(x_{2}(t)\) ist. Zeigen Sie dies durch Anwendung des Additionstheorems

    $$\cos(\alpha\pm\beta)=\cos\alpha\cos\beta\mp\sin\alpha\sin\beta.$$
    (1.193)

Ausführliche Lösungen zu den Aufgaben

1.1

  1. (a)

    Die Vektoren in (1.174) sind linear unabhängig, da (1.173) nur für \(\lambda_{1}=\lambda_{2}=\lambda_{3}=0\) erfüllt werden kann. Dies ist trivial zu sehen. Auch die Vektoren in (1.175) sind linear unabhängig. Um dies zu sehen, betrachtet man zunächst die x 3-Komponente. Sie kann nur dann verschwinden, wenn \(\lambda_{1}=0\) ist. Die x 2-Komponente verschwindet nur, wenn dann auch \(\lambda_{2}=0\) ist. Weiterhin muss auch \(\lambda_{1}=0\) erfüllt sein. Für (1.176) muss man ein lineares Gleichungssystem lösen. Es lautet

    $$\begin{aligned}-3\lambda_{1}+1\lambda_{2}+2\lambda_{3}&=0,\\ 2\lambda_{1}+2\lambda_{2}-1\lambda_{3}&=0,\\ \lambda_{1}+3\lambda_{2}-3\lambda_{3}&=0.\end{aligned}$$
    (1.194)

    Zunächst addieren wir Vielfache der zweiten Gleichung zur ersten und zur dritten, sodass jeweils \(\lambda_{3}\) verschwindet, und betrachten die beiden resultierenden Gleichungen:

    $$\begin{aligned}1\lambda_{1}+5\lambda_{2}&=0,\\ -5\lambda_{1}-3\lambda_{2}&=0.\end{aligned}$$
    (1.195)

    Wir addieren das Fünffache der ersten Gleichung zur zweiten und finden zunächst \(\lambda_{2}=0\). Durch Einsetzen in die vorherigen Gleichungssysteme ergeben sich dann \(\lambda_{1}=0\) und \(\lambda_{3}=0\). Die Vektoren sind also ebenfalls linear unabhängig. Ersetzt man allerdings den dritten Vektor durch \({\boldsymbol{v}}_{3}=(2,-4,-3)^{\top}\), ergibt sich statt (1.194) zunächst

    $$\begin{aligned}-3\lambda_{1}+1\lambda_{2}+2\lambda_{3}&=0,\\ 2\lambda_{1}+2\lambda_{2}-4\lambda_{3}&=0,\\ \lambda_{1}+3\lambda_{2}-4\lambda_{3}&=0.\end{aligned}$$
    (1.196)

    Wir addieren diesmal Vielfache der dritten Gleichung, um in den beiden anderen \(\lambda_{1}\) zu eliminieren:

    $$\begin{aligned}10\lambda_{2}-10\lambda_{3}&=0,\\ -4\lambda_{2}+4\lambda_{3}&=0.\end{aligned}$$
    (1.197)

    Diese beiden Gleichungen sind Vielfache voneinander. Sie führen somit nur auf die Bedingung \(\lambda_{2}=\lambda_{3}\). Beide Skalare können beliebige, aber identische Werte annehmen. Die drei Vektoren sind in diesem Fall also nicht linear unabhängig.

  2. (b)

    Die beiden Polynome \(p(x)\) und \(q(x)\) sind orthogonal, denn

    $$\langle p,q\rangle=\int_{-1}^{1}\left(x^{3}-2x\right)\,{\mathrm{d}}x=\left[\frac{x^{4}}{4}-x^{2}\right]_{-1}^{1}=0.$$
    (1.198)

    Dies hätte man auch direkt daran erkennen können, dass der Integrand eine ungerade Funktion von x ist, die Integrationsgrenzen aber symmetrisch sind. Allerdings sind \(p(x)\) und \(r(x)\) nicht orthogonal:

    $$\begin{aligned}\langle p,r\rangle&=\int_{-1}^{1}\left(x^{4}+x^{2}\right)\,{\mathrm{d}}x\\ &=\left[\frac{x^{5}}{5}-\frac{x^{3}}{3}\right]_{-1}^{1}=-\frac{4}{15}.\end{aligned}$$
    (1.199)

    Weiterhin findet man, dass wegen

    $$\langle q,r\rangle=\int_{-1}^{1}\left(x^{5}-x^{3}-2x\right)\,{\mathrm{d}}x=0$$
    (1.200)

    die beiden Polynome \(q(x)\) und \(r(x)\) orthogonal sind. Die Polynome lassen sich normieren, indem man beispielsweise

    $$\langle p,p\rangle=\int_{-1}^{1}x^{2}\,{\mathrm{d}}x=\frac{2}{3}$$
    (1.201)

    berechnet und entsprechend \(\tilde{p}(x):=p(x)/\sqrt{2/3}\) als normiertes Polynom definiert.

  3. (c)

    Ohne Einschränkung der Allgemeinheit können wir \({\boldsymbol{c}}=\lambda_{1}{\boldsymbol{a}}+\lambda_{2}{\boldsymbol{b}}\) schreiben. Das Spatprodukt lautet dann

    $$({\boldsymbol{a}}\times{\boldsymbol{b}})\cdot{\boldsymbol{c}}=\lambda_{1}\varepsilon_{ijk}a_{i}b_{j}a_{k}+\lambda_{2}\varepsilon_{ijk}a_{i}b_{j}b_{k}.$$
    (1.202)

    In beiden Termen auf der rechte Seite haben wir das Produkt einer antisymmetrischen Größe (Levi‐Civita‐Symbol) und einer symmetrischen Größe (\(a_{i}a_{k}\) bzw. \(b_{j}b_{k}\)) unter Vertauschung der Indizes \(i\leftrightarrow j\) bzw. \(j\leftrightarrow k\). Summiert man über alle Indizes, fallen alle Terme gegeneinander weg, denn

    $$\begin{aligned}\varepsilon_{123}a_{2}a_{3}+\varepsilon_{132}a_{3}a_{2}&=(\varepsilon_{123}+\varepsilon_{132})a_{2}a_{3}\\ &=(1-1)a_{2}a_{3}=0.\end{aligned}$$
    (1.203)
  4. (d)

    Die linke Seite von (1.180) schreiben wir zunächst in der Form

    $$\varepsilon_{kij}a_{i}b_{j}\varepsilon_{kmn}c_{m}d_{n}=\varepsilon_{kij}\varepsilon_{kmn}a_{i}b_{j}c_{m}d_{n}.$$
    (1.204)

    Hier verwendet man die Identität

    $$\varepsilon_{kij}\varepsilon_{kmn}=\delta_{im}\delta_{jn}-\delta_{in}\delta_{jm}$$
    (1.205)

    und erhält sofort

    $$\begin{aligned}({\boldsymbol{a}}\times{\boldsymbol{b}})\cdot({\boldsymbol{c}}\times{\boldsymbol{d}})&=(\delta_{im}\delta_{jn}-\delta_{in}\delta_{jm})a_{i}b_{j}c_{m}d_{n}\\ &=({\boldsymbol{a}}\cdot{\boldsymbol{c}})({\boldsymbol{b}}\cdot{\boldsymbol{d}})-({\boldsymbol{a}}\cdot{\boldsymbol{d}})({\boldsymbol{b}}\cdot{\boldsymbol{c}}).\end{aligned}$$
    (1.206)

1.2

  1. (a)

    Die allgemeinen Lösungen der Bewegungsgleichungen für das beschriebene Problem sind (siehe (1.27))

    $$x-x_{0}=v_{x,0}t,\quad z-z_{0}=v_{z,0}t-\frac{gt^{2}}{2}.$$
    (1.207)

    Die Bahnkurve erhält man, indem man t eliminiert:

    $$z-z_{0}=\frac{v_{z,0}}{v_{x,0}}(x-x_{0})-\frac{g}{2v^{2}_{x,0}}(x-x_{0})^{2}.$$
    (1.208)

    Diese ist offensichtlich eine nach unten geöffnete Parabel. Der Scheitelpunkt ist durch \({\mathrm{d}}z/{\mathrm{d}}x=0\) definiert. Dies führt auf

    $$x_{\mathrm{S}}-x_{0}=\frac{v_{x,0}v_{z,0}}{g},$$
    (1.209)

    wobei \(x_{\mathrm{S}}\) die x-Koordinate des Scheitelpunktes ist. Eingesetzt in (1.208) folgt

    $$z_{\mathrm{S}}-z_{0}=\frac{v_{z,0}^{2}}{2g}.$$
    (1.210)

    Dies ist die maximale Wurfhöhe. Dieser Punkt wird nach der Zeit

    $$t_{\mathrm{S}}=\frac{v_{z,0}}{g}$$
    (1.211)

    erreicht.

  2. (b)

    Eine einfache geometrische Überlegung führt auf

    $$v_{0}^{2}=v_{x,0}^{2}+v_{z,0}^{2},\quad v_{x,0}=v_{0}\,\cos\alpha,\quad v_{z,0}=v_{0}\,\sin\alpha.$$
    (1.212)
  3. (c)

    Um herauszufinden, wie weit der Speer fliegt, setzen wir \(z=z_{0}\). Dies führt auf zwei Lösungen. Die erste Lösung (Anfangsbedingung \(x=x_{0}\)) ist trivial. Die zweite lautet

    $$\begin{aligned}x-x_{0}&=\frac{2v_{x,0}v_{z,0}}{g}\\ &=\frac{2v_{0}^{2}}{g}\sin\alpha\,\cos\alpha=\frac{v_{0}^{2}}{g}\sin(2\alpha).\end{aligned}$$
    (1.213)

    Dies ist gerade die Weite des Wurfes. Sie wird offensichtlich für \(\alpha=\uppi/2\) maximal. Der Speer muss also genau diagonal geworfen werden, um für eine gegebene Anfangsgeschwindigkeit die größte Weite zu erreichen.

  4. (d)

    Nein, dies ist nicht möglich. Die maximale Reichweite des Fluges ist bei \(v_{0}\approx 28\,\mathrm{m}\,\mathrm{s}^{-1}\) und mit \(g\approx 10\,\mathrm{m}\,\mathrm{s}^{-2}\) ungefähr \(78\,\mathrm{m}\). Das Fußballfeld ist aber etwa \(100\,\mathrm{m}\) lang. Selbstverständlich kann der Ball noch ins Tor rollen.

1.3

  1. (a)

    Die Bewegungsgleichung in Vektorform lautet

    $$m\ddot{{\boldsymbol{x}}}=-K\dot{{\boldsymbol{x}}}-m{\boldsymbol{g}}.$$
    (1.214)

    Für die beiden relevanten Komponenten lautet sie

    $$m\ddot{x}=-K\dot{x},\quad m\ddot{z}=-K\dot{z}-mg.$$
    (1.215)

    Die Bewegungsgleichungen sind also entkoppelt.

  2. (b)

    Gleichung (1.43) beinhaltet bereits die Lösung der Bewegungsgleichung für z:

    $$\begin{aligned}z-z_{0}&=-\frac{mg}{K}t\\ &\quad+\left(v_{z,0}+\frac{mg}{K}\right)\frac{m}{K}\left[1-\exp\left(-\frac{K}{m}t\right)\right].\end{aligned}$$
    (1.216)

    Die Lösung für x erhält man, indem man g = 0 setzt und x durch z ersetzt:

    $$x-x_{0}=\frac{mv_{x,0}}{K}\left[1-\exp\left(-\frac{K}{m}t\right)\right].$$
    (1.217)
  3. (c)

    Man erkennt, dass die Zeit t in (1.216) an zwei Stellen vorkommt: einmal als linearer Term, das zweite Mal innerhalb der Exponentialfunktion. Da die Exponentialfunktionen in den Lösungen für \(x(t)\) und \(z(t)\) identisch sind, kann man zunächst

    $$z-z_{0}=-\frac{mg}{K}t+\left(v_{z,0}+\frac{mg}{K}\right)\frac{x-x_{0}}{v_{x,0}}$$
    (1.218)

    schreiben. Es verbleibt noch die Zeit im linearen Term. Hierzu lösen wir (1.217) nach t auf,

    $$t=-\frac{m}{K}\ln\left[1-\frac{K}{mv_{x,0}}(x-x_{0})\right],$$
    (1.219)

    und setzen das Ergebnis in (1.218) ein:

    $$\begin{aligned}z-z_{0}&=\frac{m^{2}g}{K^{2}}\ln\left[1-\frac{K(x-x_{0})}{mv_{x,0}}\right]\\ &\quad+\left(v_{z,0}+\frac{mg}{K}\right)\frac{x-x_{0}}{v_{x,0}}.\end{aligned}$$
    (1.220)
  4. (d)

    Wir wollen den Grenzfall \(K\to 0\) untersuchen. Einsetzen von K = 0 führt auf Probleme, daher muss zunächst der Logarithmus entwickelt werden. Für \(K\to 0\) ist die folgende Näherung gültig (indem man um x = 0 entwickelt):

    $$\begin{aligned}&\ln\left[1-\frac{K(x-x_{0})}{mv_{x,0}}\right]\\ &\quad\approx-\frac{K(x-x_{0})}{mv_{x,0}}-\frac{K^{2}(x-x_{0})^{2}}{2m^{2}v^{2}_{x,0}}.\end{aligned}$$
    (1.221)

    Setzt man (1.221) in (1.220) ein, folgt sofort

    $$z-z_{0}=\frac{v_{z,0}}{v_{x,0}}(x-x_{0})-\frac{g}{2v^{2}_{x,0}}(x-x_{0})^{2}.$$
    (1.222)

    Dies ist genau (1.208), die schon beim schiefen Wurf ohne Reibung gefunden wurde. Da der Faktor vor dem Logarithmus in (1.220) K 2 im Nenner enthält, muss im Zähler mindestens bis zur zweiten Ordnung entwickelt werden, damit kein K im Nenner übrig bleibt. Würde man höhere Terme der Entwicklung des Logarithmus berücksichtigen, würden diese wegen \(K\to 0\) keine Rolle spielen.

1.4

  1. (a)

    Aus (1.181) folgt

    $$\ddot{{\boldsymbol{x}}}=\frac{q}{m}\dot{{\boldsymbol{x}}}\times{\boldsymbol{B}}.$$
    (1.223)

    Somit steht der Beschleunigungsvektor stets senkrecht auf dem Geschwindigkeitsvektor. Es gibt keine Beschleunigungskomponente entlang der Bahnkurve bzw. entlang des Tangentialvektors \({\boldsymbol{\tau}}\). Wegen (1.124) und (1.125) bedeutet dies aber gerade, dass der Betrag der Bahngeschwindigkeit konstant ist. Gleichung (1.181) besagt weiterhin, dass die Kraft \({\boldsymbol{F}}\) und die Geschwindigkeit \(\dot{{\boldsymbol{x}}}\) senkrecht aufeinander stehen. Ihr Skalarprodukt verschwindet somit. Wegen (1.149) ist die Energie erhalten. Es wird also keine Arbeit am Teilchen verrichtet.

  2. (b)

    Zunächst ist das Kreuzprodukt auszuwerten. Mit

    $$({\boldsymbol{v}}\times{\boldsymbol{B}})_{i}=\varepsilon_{ijk}v_{j}B_{k}$$
    (1.224)

    und (1.182) folgt

    $$\ddot{x}=\frac{qB}{m}\dot{y},\quad\ddot{y}=-\frac{qB}{m}\dot{x},\quad\ddot{z}=0.$$
    (1.225)

    Unter Berücksichtigung der Anfangsbedingungen lautet die Lösung für die Bewegung entlang der z-Achse einfach

    $$z=v_{z,0}t.$$
    (1.226)
  3. (c)

    Die Bewegungsgleichungen für x und y sind noch gekoppelt. Wir integrieren zunächst die Gleichung für x einmal nach der Zeit. Wegen \(\dot{x}(t=0)=0\) und \(y(t=0)=0\) folgt

    $$\dot{x}=\frac{qB}{m}y.$$
    (1.227)

    Dieses Ergebnis kann in die Bewegungsgleichung für y eingesetzt werden:

    $$\ddot{y}=-\omega_{B}^{2}y.$$
    (1.228)

    Hier wurde

    $$\omega_{B}:=\frac{qB}{m}$$
    (1.229)

    als Abkürzung definiert. Man nennt \(\omega_{B}\) die Zyklotronfrequenz. Die allgemeine Lösung von (1.228) kann in der Form

    $$y=A_{y}\sin(\omega_{B}t-\phi_{y})$$
    (1.230)

    geschrieben werden. Die beiden Integrationskonstanten ergeben sich aus den Anfangsbedingungen. Man kann schnell nachprüfen, dass \(\phi_{y}=0\) aus \(y(t=0)=0\) folgt. Weiterhin ist \(A_{y}=v_{y,0}/\omega_{B}\). Dies erkennt man, wenn man die erste Zeitableitung von (1.230) berechnet und mit der Anfangsbedingung \(\dot{y}(t=0)=v_{y,0}\) vergleicht. Der letzte Schritt ist die Integration von (1.227), in die zuvor (1.230) eingesetzt wird:

    $$\begin{aligned}x&=-\frac{qB}{m\omega_{B}}A_{y}\,\cos(\omega_{B}t)+C_{1}\\ &=-\frac{v_{y,0}}{\omega_{B}}\cos(\omega_{B}t)+C_{1}.\end{aligned}$$
    (1.231)

    Die Anfangsgeschwindigkeit \(\dot{x}(t=0)=0\) ist automatisch erfüllt, da sie bereits für (1.227) verwendet wurde. Die letzte verbleibende Integrationskonstante, C 1, ergibt sich aus der letzten Forderung: \(x(t=0)=0\). Die Lösung der Bewegungsgleichung lautet zusammengefasst

    $$\begin{aligned}x&=\frac{v_{y,0}}{\omega_{B}}\left[1-\cos(\omega_{B}t)\right],\\ y&=\frac{v_{y,0}}{\omega_{B}}\sin(\omega_{B}t),\\ z&=v_{z,0}t.\end{aligned}$$
    (1.232)

    Das Teilchen bewegt sich auf einer Schraubenlinie ähnlich zu der in Abb. 1.11. Der Unterschied hier ist, dass der Ursprung nicht auf der Achse der Schraubenlinie liegt. Er ist um \(v_{y,0}/\omega_{B}\) entlang der x-Achse verschoben.

  4. (d)

    Zunächst berechnen wir die Geschwindigkeit:

    $$\dot{{\boldsymbol{x}}}=\begin{pmatrix}-v_{y,0}\,\sin(\omega_{B}t)\\ v_{y,0}\,\cos(\omega_{B}t)\\ v_{z,0}\end{pmatrix}.$$
    (1.233)

    Ihr Betrag ist \(v:=\sqrt{v_{y,0}^{2}+v_{z,0}^{2}}={\mathrm{const}}\). Der Tangentialvektor erfüllt wegen (1.124)

    $${\boldsymbol{\tau}}=\frac{\dot{{\boldsymbol{x}}}}{\lvert\dot{{\boldsymbol{x}}}\rvert}=\frac{1}{v}\begin{pmatrix}-v_{y,0}\,\sin(\omega_{B}t)\\ v_{y,0}\,\cos(\omega_{B}t)\\ v_{z,0}\end{pmatrix}.$$
    (1.234)

    Der Bahnparameter und die Zeit sind demnach über

    $$\frac{{\mathrm{d}}s}{{\mathrm{d}}t}=v$$
    (1.235)

    verbunden, da wegen der Definition des Tangentialvektors \(\lvert{\boldsymbol{\tau}}\rvert=\lvert{\mathrm{d}}{\boldsymbol{x}}/{\mathrm{d}}s\rvert=1\) erfüllt sein muss. Der Hauptnormalenvektor aus (1.114) ist

    $$\begin{aligned}{\boldsymbol{n}}_{\mathrm{H}}&=\frac{{\mathrm{d}}{\boldsymbol{\tau}}}{{\mathrm{d}}s}\left\lvert\frac{{\mathrm{d}}{\boldsymbol{\tau}}}{{\mathrm{d}}s}\right\rvert^{-1}=\frac{{\mathrm{d}}{\boldsymbol{\tau}}}{{\mathrm{d}}t}\left\lvert\frac{{\mathrm{d}}{\boldsymbol{\tau}}}{{\mathrm{d}}t}\right\rvert^{-1}\\ &=\begin{pmatrix}\cos(\omega_{B}t)\\ -\sin(\omega_{B}t)\\ 0\end{pmatrix}.\end{aligned}$$
    (1.236)

    Schließlich fehlt noch der Krümmungsradius, der wegen (1.123) den Wert

    $$\rho=\left\lvert\frac{{\mathrm{d}}{\boldsymbol{\tau}}}{{\mathrm{d}}s}\right\rvert^{-1}=\left\lvert\frac{{\mathrm{d}}{\boldsymbol{\tau}}}{{\mathrm{d}}t}\frac{{\mathrm{d}}t}{{\mathrm{d}}s}\right\rvert^{-1}=\frac{v^{2}}{v_{y,0}\omega_{B}}=\frac{mv^{2}}{qv_{y,0}B}$$
    (1.237)

    annimmt.

1.5

  1. (a)

    In Indexschreibweise lautet die Rotation des Kraftfeldes

    $$\left({\boldsymbol{\nabla}}\times\left[F(r)\frac{{\boldsymbol{r}}}{r}\right]\right)_{i}=\varepsilon_{ijk}\nabla_{j}F(r)\frac{x_{k}}{r}.$$
    (1.238)

    Statt \(\nabla_{j}\) könnte man genauso gut \(\partial_{j}\) schreiben; beide Terme sind identisch. Es ist unbedingt zu beachten, dass die Ableitung \(\nabla_{j}\) auf alle Terme wirkt, die rechts von ihr stehen. Als Hilfsgleichung verwenden wir

    $$\nabla_{j}r=\nabla_{j}\sqrt{x_{1}^{2}+x_{2}^{2}+x_{3}^{2}}=\frac{x_{j}}{\sqrt{x_{1}^{2}+x_{2}^{2}+x_{3}^{2}}}=\frac{x_{j}}{r}.$$
    (1.239)

    Wir wenden zunächst die Produktregel an und sehen, dass wir es mit drei Faktoren zu tun haben, die wir im Einzelnen untersuchen wollen:

    $$\begin{aligned}\frac{x_{k}}{r}\nabla_{j}F(r)&=\frac{x_{j}x_{k}}{r^{2}}\frac{{\mathrm{d}}F(r)}{{\mathrm{d}}r},\\ \frac{F(r)}{r}\nabla_{j}x_{k}&=\frac{F(r)}{r}\delta_{jk},\\ F(r)x_{k}\nabla_{j}\frac{1}{r}&=-\frac{x_{j}x_{k}}{r^{3}}F(r).\end{aligned}$$
    (1.240)

    Bei der Summation über j und k treten die beiden Kombinationen \(\varepsilon_{ijk}\delta_{jk}\) und \(\varepsilon_{ijk}x_{j}x_{k}\) auf. Sowohl \(\delta_{jk}\) als auch \(x_{j}x_{k}\) sind symmetrisch unter Vertauschung der beiden Indizes, aber \(\varepsilon_{ijk}=-\varepsilon_{ikj}\) ist antisymmetrisch. Bei der Summation fallen daher alle Terme mit \(x_{j}x_{k}\) paarweise gegeneinander weg:

    $$\varepsilon_{ijk}x_{j}x_{k}+\varepsilon_{ikj}x_{k}x_{j}=\varepsilon_{ijk}x_{j}x_{k}-\varepsilon_{ijk}x_{j}x_{k}=0.$$
    (1.241)

    Die Kontraktion \(\varepsilon_{ijk}\delta_{jk}\) verschwindet, da \(\varepsilon_{ijk}\) nur für \(j\not=k\), \(\delta_{jk}\) allerdings nur für \(j=k\) ungleich null ist. Damit ist gezeigt, dass die Rotation von \({\boldsymbol{F}}({\boldsymbol{r}})\) verschwindet.

  2. (b)

    Man erkennt an der Form der Kraft, dass alle Ableitungen, die F 3 oder x 3 betreffen, verschwinden. Somit reduziert sich die Rotation direkt auf

    $${\boldsymbol{\nabla}}\times{\boldsymbol{F}}({\boldsymbol{r}})={\boldsymbol{\hat{e}}}_{3}\left(\frac{\partial F_{2}}{\partial x_{1}}-\frac{\partial F_{1}}{\partial x_{2}}\right).$$
    (1.242)

    Aus

    $$\frac{\partial\varrho}{\partial x_{1}}=\frac{x_{1}}{\varrho},\quad\frac{\partial\varrho}{\partial x_{2}}=\frac{x_{2}}{\varrho}$$
    (1.243)

    folgt weiterhin, dass sich die beiden verbleibenden Ableitungen in (1.242) gegenseitig aufheben. Somit ist auch hier das Kraftfeld rotationsfrei. Das Kurvenintegral lautet

    $$\oint{\boldsymbol{F}}({\boldsymbol{r}})\cdot{\mathbf{d}}{\boldsymbol{r}}=\int_{0}^{2\uppi}{\boldsymbol{F}}({\boldsymbol{r}}(\varphi))\cdot\frac{{\mathrm{d}}{\boldsymbol{r}}}{{\mathrm{d}}\varphi}{\mathrm{d}}\varphi.$$
    (1.244)

    Die Kraft muss als Funktion des Winkels \(\varphi\) geschrieben werden. Dies ist möglich, indem x 1 und x 2 durch die entsprechenden Ausdrücke in (1.186) ersetzt werden:

    $${\boldsymbol{F}}({\boldsymbol{r}}(\varphi))=\frac{1}{R}\begin{pmatrix}-\sin\varphi\\ \cos\varphi\\ 0\end{pmatrix}.$$
    (1.245)

    Gleichung (1.244) lautet dann

    $$\oint{\boldsymbol{F}}({\boldsymbol{r}})\cdot{\mathbf{d}}{\boldsymbol{r}}=\int_{0}^{2\uppi}\left(\sin^{2}\varphi+\cos^{2}\varphi\right)\,{\mathrm{d}}\varphi=2\uppi\not=0.$$
    (1.246)

    Wählt man einen Weg, der um den Pol bei \(x_{1}=x_{2}=0\) herumläuft, verschwindet das geschlossene Wegintegral nicht, obwohl die Rotation des Kraftfeldes null ist. Die Erklärung ist, dass das Kraftfeld nicht einfach zusammenhängend ist. In der Tat gibt es bei \(x_{1}=x_{2}=0\) eine Definitionslücke, die das beobachtete Verhalten erklärt.

  3. (c)

    Die Richtungsableitung entlang einer Achse \({\boldsymbol{\hat{n}}}\) ist definert als

    $$\partial_{{\boldsymbol{\hat{n}}}}g({\boldsymbol{x}})={\boldsymbol{\hat{n}}}\cdot{\boldsymbol{\nabla}}g({\boldsymbol{x}})=\hat{n}_{i}\partial_{i}g({\boldsymbol{x}}).$$
    (1.247)

    Für \({\boldsymbol{\hat{n}}}={\boldsymbol{\hat{e}}}_{j}\) ist offensichtlich \(\hat{e}_{j,i}\partial_{i}=\delta_{ij}\partial_{i}=\partial_{j}\) und somit

    $$\partial_{{\boldsymbol{\hat{n}}}_{j}}g({\boldsymbol{x}})=\partial_{j}g({\boldsymbol{x}}),$$
    (1.248)

    was zu zeigen war.

  4. (d)

    Wir schreiben die Divergenz in Komponenten und sortieren um:

    $${\boldsymbol{\nabla}}\cdot\left({\boldsymbol{\nabla}}\times{\boldsymbol{h}}({\boldsymbol{x}})\right)=\partial_{i}\epsilon_{ijk}\partial_{j}h_{k}=\epsilon_{ijk}\partial_{i}\partial_{j}h_{k}.$$
    (1.249)

    Dieser Ausdruck muss verschwinden, wenn die beiden Ableitungen vertauschbar sind, da über symmetrische (\(\partial_{i}\partial_{j}\)) und antisymmetrische (\(\epsilon_{ijk}\)) Terme summiert wird.

  5. (e)

    Zunächst ist

    $$\begin{aligned}{\boldsymbol{\nabla}}\times\left({\boldsymbol{\nabla}}\times{\boldsymbol{h}}({\boldsymbol{x}})\right)&=\epsilon_{ijk}{\boldsymbol{\hat{e}}}_{i}\partial_{j}\left(\epsilon_{klm}\partial_{l}h_{m}\right)\\ &=\epsilon_{ijk}\epsilon_{klm}{\boldsymbol{\hat{e}}}_{i}\partial_{j}\partial_{l}h_{m}.\end{aligned}$$
    (1.250)

    Hier nutzt man das bekannte Ergebnis

    $$\epsilon_{ijk}\epsilon_{klm}=\epsilon_{kij}\epsilon_{klm}=\delta_{il}\delta_{jm}-\delta_{im}\delta_{jl}$$
    (1.251)

    aus. Es folgt

    $$\begin{aligned}{\boldsymbol{\nabla}}\times\left({\boldsymbol{\nabla}}\times{\boldsymbol{h}}({\boldsymbol{x}})\right)&={\boldsymbol{\hat{e}}}_{l}\partial_{l}\partial_{m}h_{m}-{\boldsymbol{\hat{e}}}_{m}\partial_{l}\partial_{l}h_{m}\\ &={\mathbf{grad}}\,{\mathrm{div}}\,{\boldsymbol{h}}-(\partial_{l}\partial_{l}){\boldsymbol{h}}.\end{aligned}$$
    (1.252)

    Der sogenannte Laplace‐Operator

    $$\Updelta:=\partial_{l}\partial_{l}$$
    (1.253)

    wird uns in Kap. 2 wieder begegnen.

1.6

  1. (a)

    Wir berechnen die Rotation von \({\boldsymbol{F}}({\boldsymbol{x}})\):

    $$\begin{aligned}{\boldsymbol{\nabla}}\times{\boldsymbol{F}}=\epsilon_{ijk}{\boldsymbol{\hat{e}}}_{i}\partial_{j}F_{k}&=C\begin{pmatrix}x_{3}-2k_{2}x_{3}\\ 0-0\\ 2x_{1}-k_{1}x_{1}\end{pmatrix}\\ &=C\begin{pmatrix}(1-2k_{2})x_{3}\\ 0\\ (2-k_{1})x_{1}\end{pmatrix}.\end{aligned}$$
    (1.254)

    Die Rotation verschwindet für \(k_{1}=2\) und \(k_{2}=\frac{1}{2}\). Da das Kraftfeld einfach zusammenhängend ist (es gibt keine Definitionslücken), muss es dann ein Potenzial \(V({\boldsymbol{x}})\) geben.

  2. (b)

    Der einfachste Weg in kartesischen Koordinaten ist eine Gerade mit der Parametrisierung

    $${\boldsymbol{x}}(s)=s{\boldsymbol{a}},\quad s\in[0,1].$$
    (1.255)

    Für s = 0 und s = 1 befindet man sich jeweils am Anfangspunkt (Ursprung O) und Endpunkt (\({\boldsymbol{a}}\)). Selbstverständlich könnte man auch entlang der einzelnen Achsen integrieren oder irgendeinen anderen Weg verfolgen. Die geleistete Arbeit lautet

    $$W=\int_{O}^{\boldsymbol{a}}{\boldsymbol{F}}({\boldsymbol{x}})\cdot{\mathbf{d}}{\boldsymbol{x}}=\int_{0}^{1}{\boldsymbol{F}}({\boldsymbol{x}}(s))\frac{{\mathbf{d}}{\boldsymbol{x}}(s)}{{\mathrm{d}}s}{\mathrm{d}}s$$
    (1.256)

    mit der Ableitung \({\mathbf{d}}{\boldsymbol{x}}(s)/{\mathrm{d}}s={\boldsymbol{a}}\). Da s die Integrationsvariable ist, müssen die Koordinaten \({\boldsymbol{x}}\) durch s ausgedrückt werden. Dies ist aber mit (1.255) bereits geschehen. Wir verwenden also \(x_{i}=a_{i}s\) (i = 1,2,3). Zu lösen ist somit das Integral

    $$W =C\int_{0}^{1}\left[2a_{1}^{2}a_{2}+\left(a_{1}^{2}a_{2}+a_{2}a_{3}^{2}/2\right)+a_{2}a_{3}^{2}\right]s^{2}\,{\mathrm{d}}s$$
    $$ =C\left(3a_{1}^{2}+3a_{3}^{2}/2\right)a_{2}\int_{0}^{1}s^{2}\,{\mathrm{d}}s$$
    $$ =Ca_{2}\left(a_{1}^{2}+a_{3}^{2}/2\right).$$
    (1.257)

    Die geleistete Arbeit ist je nach Vorzeichen von a 2 positiv oder negativ. Ist \(a_{2}=0\), wird gar keine Energie umgesetzt.

  3. (c)

    Das Potenzial erhält man direkt aus (1.257). Dazu muss man sich überlegen, dass das Potenzial am Ort \({\boldsymbol{x}}\) die negative Arbeit ist, die man benötigt, um eine Punktmasse von einem Bezugspunkt nach \({\boldsymbol{x}}\) zu bewegen. Wählt man als Bezugspunkt den Ursprung und benennt \({\boldsymbol{a}}\) in \({\boldsymbol{x}}\) um, lautet das Potenzial

    $$V({\boldsymbol{x}})=-Cx_{2}\left(x_{1}^{2}+x_{3}^{2}/2\right).$$
    (1.258)

    Es soll zum Abschluss überprüft werden, ob sich die korrekte Kraft daraus ergibt:

    $$\begin{aligned}{\boldsymbol{F}}({\boldsymbol{x}})=-{\boldsymbol{\nabla}}V({\boldsymbol{x}})&=-\begin{pmatrix}\partial_{1}V({\boldsymbol{x}})\\ \partial_{2}V({\boldsymbol{x}})\\ \partial_{3}V({\boldsymbol{x}})\end{pmatrix}\\ &=\begin{pmatrix}2x_{1}x_{2}\\ x_{1}^{2}+x_{3}^{2}/2\\ x_{2}x_{3}\end{pmatrix}.\end{aligned}$$
    (1.259)

    Dies ist offensichtlich das richtige Ergebnis.

1.7

  1. (a)

    Wir berechnen zunächst die zweite Zeitableitung der Potenzreihe:

    $$\begin{aligned}\dot{x}(t)&=\sum_{j=1}^{\infty}j\omega_{0}c_{j}(\omega_{0}t)^{j-1},\\ \ddot{x}(t)&=\sum_{j=2}^{\infty}j(j-1)\omega_{0}^{2}c_{j}(\omega_{0}t)^{j-2}.\end{aligned}$$
    (1.260)

    Es wurde dabei berücksichtigt, dass die Ableitung von t 0 verschwindet. In der zweiten Zeitableitung wird der Index verschoben, sodass die Summe wieder bei j = 0 beginnt:

    $$\ddot{x}(t)=\sum_{j=0}^{\infty}(j+2)(j+1)\omega_{0}^{2}c_{j+2}(\omega_{0}t)^{j}.$$
    (1.261)

    Durch Einsetzen der Potenzreihe (1.190) in die Schwingungsgleichung (1.189) folgt

    $$\sum_{j=0}^{\infty}\left[(j+2)(j+1)c_{j+2}+c_{j}\right](\omega_{0}t)^{i}=0.$$
    (1.262)

    Damit diese Gleichung zu allen Zeiten erfüllt werden kann, müssen alle Summanden einzeln verschwinden. Dies führt auf die Rekursionsgleichung

    $$c_{j+2}=-\frac{c_{j}}{(j+2)(j+1)}.$$
    (1.263)

    Für bekanntes c 0 sind alle c j mit geraden j gegeben. Genauso folgen aus bekanntem c 1 alle c j mit ungeraden j. Somit sind nur c 0 und c 1 nicht festgelegt. Die Wahl dieser beiden Koeffizienten entspricht den beiden Anfangsbedingungen des Problems, da es sich um eine Differenzialgleichung zweiter Ordnung handelt.

  2. (b)

    Die explizite Form soll nicht mit mathematischer Strenge gezeigt werden (man könnte sie jedoch mit vollständiger Induktion beweisen). Zunächst nehmen wir an, c 0 sei bekannt. Dann folgt sofort \(c_{2}=-c_{0}/(1\cdot 2)\), daraus wiederum \(c_{4}=+c_{0}/(1\cdot 2\cdot 3\cdot 4)\) usw. Offenbar wechselt das Vorzeichen von Iteration zu Iteration, und im Nenner steht der Term \(j!\). Analog findet man den expliziten Ausdruck, wenn man mit c 1 beginnt. Die Exponenten in (1.192) stellen sicher, dass die Vorzeichen stimmen.

  3. (c)

    Um die Taylor‐Entwicklung von Sinus und Kosinus zu erhalten, müssen die ersten Ableitungen berechnet werden. Es ist

    $$\begin{aligned}\frac{{\mathrm{d}}\cos(\omega_{0}t)}{{\mathrm{d}}t}&=-\omega_{0}\sin(\omega_{0}t),\\ \frac{{\mathrm{d}}^{2}\cos(\omega_{0}t)}{{\mathrm{d}}t^{2}}&=-\omega_{0}^{2}\cos(\omega_{0}t),\\ \frac{{\mathrm{d}}^{3}\cos(\omega_{0}t)}{{\mathrm{d}}t^{3}}&=+\omega_{0}^{3}\sin(\omega_{0}t),\\ \frac{{\mathrm{d}}^{4}\cos(\omega_{0}t)}{{\mathrm{d}}t^{4}}&=+\omega_{0}^{4}\cos(\omega_{0}t).\end{aligned}$$
    (1.264)

    Anwendung der Taylor’schen Formel liefert

    $$\cos(\omega_{0}t)=\frac{1}{0!}(\omega_{0}t)^{0}-\frac{1}{2!}(\omega_{0}t)^{2}+\frac{1}{4!}(\omega_{0}t)^{4}\mp\ldots$$
    (1.265)

    Für den Sinus findet man analog

    $$\sin(\omega_{0}t)=\frac{1}{1!}(\omega_{0}t)^{1}-\frac{1}{3!}(\omega_{0}t)^{3}+\frac{1}{5!}(\omega_{0}t)^{5}\mp\ldots$$
    (1.266)

    Tatsächlich entspricht die Reihenentwicklung des Kosinus genau der Entwicklung (1.190) für gerade j und die des Sinus der Entwicklung für ungerade j. Man kann daraus folgern, dass sowohl der Kosinus als auch der Sinus die Schwingungsgleichung lösen.

  4. (d)

    Wegen

    $$\begin{aligned}\ddot{x}_{1}(t)&=-\omega_{0}^{2}\cos(\omega_{0}t)=-\omega_{0}^{2}x_{1}(t),\\ \ddot{x}_{2}(t)&=-\omega_{0}^{2}\sin(\omega_{0}t)=-\omega_{0}^{2}x_{2}(t)\end{aligned}$$
    (1.267)

    sind sowohl \(x_{1}(t)\) als auch \(x_{2}(t)\) Lösungen der Schwingungsgleichung. Es ist noch zu zeigen, dass

    $$A\cos(\omega_{0}t-\phi_{0})=A_{1}\cos(\omega_{0}t)+A_{2}\sin(\omega_{0}t)$$
    (1.268)

    gilt. Dabei folgen A und \(\phi_{0}\) aus A 1 und A 2. Als Anfangsbedingungen sind beide Paare gleichwertig. Das Additionstheorem (1.193) bedeutet im vorliegenden Fall

    $$\begin{aligned}&A\cos(\omega_{0}t-\phi_{0})\\ &=(A\cos\phi_{0})\cos(\omega_{0}t)+(A\sin\phi_{0})\sin(\omega_{0}t).\end{aligned}$$
    (1.269)

    Offensichtlich haben wir das Ziel mit \(A_{1}=A\cos\phi_{0}\) und \(A_{2}=A\sin\phi_{0}\) erreicht.

Antworten zu den Selbstfragen

Antwort 9

Die Beschleunigung ist etwa um einen Faktor 2,7 größer.

Antwort 17

Die Radialbeschleunigung muss größer sein als die Gravitationsbeschleunigung, damit die Flüssigkeit am höchsten Punkt gegen den Boden des Eimers gedrückt wird.

Rights and permissions

Reprints and permissions

Copyright information

© 2015 Springer-Verlag Berlin Heidelberg

About this chapter

Cite this chapter

Bartelmann, M., Feuerbacher, B., Krüger, T., Lüst, D., Rebhan, A., Wipf, A. (2015). Die Newton’schen Axiome. In: Theoretische Physik. Springer Spektrum, Berlin, Heidelberg. https://doi.org/10.1007/978-3-642-54618-1_1

Download citation

Publish with us

Policies and ethics